Top Banner
Contracts II – Outline I. Supplementing the Agreement A. Gap Fillers 1. Generally – a. Cts implicit agreements through: i. the course of performance – post-K-formation behavior; “the actual performance tendered and accepted w/o objection is a strong indicator of what must have been intended – UCC § 2.208 & R2K § 202(4) ii. course of dealings – any relationship the parties may have had in the period before the transaction in question – only pertinent if the earlier relationship is comparable or analogous - UCC § 1.205 (1) & R2K §§ 202, 203 iii. trade usages – a well-accepted custom or practice that explains language or supplements an omission in the agreement – helpful to ascertain the parties’ intent – UCC § 1.205 & R2K § 202(5) b. Gap Fillers are used to supplement K, not to override the parties’ probable intent; parties can “contract out” of a legally implied standard term by expressing contrary intent 2. Gap Fillers—Ct will apply a “reasonable” value for the given term: a. UCC Article 2 – i. §§ 2.312, 2.314, 2.315 – imply certain minimum warranties that a seller makes under defined circumstances regarding the title to and quality of the goods; ii. § 2.305 – if no price is stipulated, a “reasonable price” at the time of delivery, unless the apparent intent of the agreement is otherwise, and in “good faith” iii. § 2.309 – time of delivery, notice of termination – OK to terminate is no reasonable detriment; if goods are specific, cannot terminate; if seller stocked goods for buyer, cannot terminate iv. § 2.308 – place of delivery v. § 2.310 – time of payment vi. § 2.509 – risk of loss vii. § 2.513 – buyer’s right of inspection B. Warranties in Art. 2 1. Express Warranties – UCC § 2.313
52
Welcome message from author
This document is posted to help you gain knowledge. Please leave a comment to let me know what you think about it! Share it to your friends and learn new things together.
Transcript
Page 1: Contracts II – Outline

Contracts II – OutlineI. Supplementing the Agreement

A. Gap Fillers1. Generally –

a. Cts implicit agreements through: i. the course of performance – post-K-formation behavior; “the actual performance tendered and accepted w/o objection is a strong indicator of what must have been intended – UCC § 2.208 & R2K § 202(4) ii. course of dealings – any relationship the parties may have had in the period before the transaction in question – only pertinent if the earlier relationship is comparable or analogous - UCC § 1.205 (1) & R2K §§ 202, 203iii. trade usages – a well-accepted custom or practice that explains language or supplements an omission in the agreement – helpful to ascertain the parties’ intent – UCC § 1.205 & R2K § 202(5)

b. Gap Fillers are used to supplement K, not to override the parties’ probable intent; parties can “contract out” of a legally implied standard term by expressing contrary intent

2. Gap Fillers—Ct will apply a “reasonable” value for the given term:a. UCC Article 2 –

i. §§ 2.312, 2.314, 2.315 – imply certain minimum warranties that a seller makes under defined circumstances regarding the title to and quality of the goods;ii. § 2.305 – if no price is stipulated, a “reasonable price” at the time of delivery, unless the apparent intent of the agreement is otherwise, and in “good faith”iii. § 2.309 – time of delivery, notice of termination – OK to terminate is no reasonable detriment; if goods are specific, cannot terminate; if seller stocked goods for buyer, cannot terminateiv. § 2.308 – place of deliveryv. § 2.310 – time of paymentvi. § 2.509 – risk of lossvii. § 2.513 – buyer’s right of inspection

B. Warranties in Art. 21. Express Warranties – UCC § 2.313

a. “An affirmation of fact or promise by the seller to the buyer which relates to the goods and becomes part of the basis of the bargain creates an express warranty that the goods shall

conform to the affirmation or promise.”ii. A description of the goods creates a warranty that they will conform to the description iii. A sample or model of the goods creates a warranty that the “whole of the goods” will conform to the sample or model

b. An affirmation merely of the value of the goods or a statement purporting to be merely the seller’s opinion or commendation of the goods does not create a warranty – “sales talk” or

“puffery”2. Implied Warranty of Merchantability – UCC § 2.314

a. “Unless excluded or modified …a warranty that goods shall be merchantable is implied in a K for their sale if the seller is a merchant w/ respect to goods of that kind.”b. Merchantable: goods must be “fit for the ordinary purposes for which such goods are used”c. Always becomes part of the K unless it expressly disclaims themd. Koken v. Black & Veatech Construction, Inc. –

i. The question is not the subjective expectations of the particular user, but the reasonable expectations of an ordinary user or purchaserii. Centered around reasonable consumer expectations; standard is objective rather than subjective

Page 2: Contracts II – Outline

3. Implied Warranty of Fitness for Particular Purpose – UCC § 2.315a. Not limited to Merchants: “Where the seller at the time of contracting has reason to know any particular purpose for which the goods are required and that the buyer is relying on the seller’s judgment to select or furnish suitable goods, there is…an implied warranty that the goods shall be fit for such purpose.”b. Elements:

i. The seller had reason to know of the buyer’s purposeii. The seller had reason to know that the buyer was relying on the seller’s skill or judgment to furnish suitable goods; and iii. The buyer did in fact rely on the seller’s skill or judgment iv. If buyer insists on a particular brand of goods, he is not relying on the seller’s skill or judgment, so no implied warranty of fitness for a particular purpose arises

c. Lewis v. Mobil Oil Corp. - i. The buyer need not bring home to the seller actual knowledge of the particular purpose for which the goods are intended or of his reliance on the seller’s skill and judgment, if the circumstances are such that the seller has reason to realize the purpose intended or that the reliance existsii. If Mobil needed more information to make a proper determination, it was incumbent on Mobil to get the additional information before making its recommendation

4. Disclaiming Warranties from the K – UCC § 2.316a. Express Warranties: Free to disclaim; does so in a clear and reasonable way; rarely makes one, so rarely needs to disclaimb. Implied Warranty of Merchantability: MUST mention the word “merchantability” and if written must be conspicuous – “As is” disclaimer must also be conspicuousc. Implied Warranty of Fitness for Particular Purpose: must be in a conspicuous writing –d. When a buyer has inspected goods before entering into a K, there are no implied warranties w/ respect to defects that ought to have been revealed by inspectione. South Carolina Electric & Gas Co. v. Combustion Engineering, Inc. –

i. A disclaimer of implied warranties for goods will be upheld even though the disclaimer does not meet statutory requirements for how it must be written and presented in a K where the parties are commercially sophisticated and the course of dealing indicates that the language of the disclaimer was neither unbargained for nor unexpected by the purchaser

f. Henningsen v. Bloomfield Motors, Inc i. A warranty disclaimer which deprives the consumer of the protection which the “legislative

will” has sought to provide through implied warranties, clearly tends toward the injury of the publicii. Provisions that clearly tend toward the injury of the public will be declared void as against public policyiv. Ct came out against an “exculpatory clause” in an adhesion K squarely because it violated PP

II. Limits on the Bargain and its PerformanceA. Unfairness

1. Cts have manipulated the doctrine of “consideration” to promote fairness in dealing; and find that a particularly overbearing provision conflicts with an established public policy

a. Conflict between paternalism v. freedom of K2. Usually Cts don’t look at value of the consideration, only if it is there –

a. However, in a Ct of equity, a Ct will look to the value of the consideration for “fairness” – i. Fairness is measured at the time the K is madeii. Especially if parties are not dealing “at arms length”

b. Risk – when a party does not understand what it is giving up, a Ct may determine that K is unfair, especially in equity

Page 3: Contracts II – Outline

3. Cts of equitya. McKinnon v. Benedict

i. D, in order to buy a summer resort, borrowed $5000 from adjoining property owner, P, who promised to help them attract tourists if Ds would not cut trees between the resort and his property nor make certain other improvements for a period of 25 years.ii. K which are oppressive; fail to meet the test of reasonableness, will not be enforced in equity

(a) The K was oppressive: Ds being of very weak bargaining power next to P, received inadequate consideration for the severe restriction he promised to uphold(b) Sophistication of parties is important to Ct’s decision

iii. Restrictions and prohibitions as to the use of real estate should be resolved, if a doubt exists, in favor of the free use of the property, because of the principle of public policy that restrictions on the use of land “are not favored by the law”iv. Although a K is harsh, oppressive, and unconscionable, it may nevertheless be enforceable at law; but, in the discretion of the Ct, equitable remedies will not be enforced against one who suffers from such harshness and oppression

b. Tuckwiller v. Tuckwiller i. Prior services cannot provide the consideration essential to a binding K. However, prior services and the past relation of the parties may properly be considered in connection with the fairness of the K and the adequacy of the considerationii. Whenever a K concerning real property is in its nature and incidents entirely unobjectionable – that is, when it possesses none of those features which…appeal to the discretion of the Ct – it is an much a matter of course of a Ct of equity to decree a SP of it, as it is for a Ct of law to give damages for breach of it.

(a) The surrounding circumstances in equity actions become important in guiding the Ct to its decision. (b) Because of the lack of relatives of Mrs. Morrison, her satisfaction expressed by her actions, plus in some respects the fact Mrs. Tuckwiller quit her job to fulfill her side of the bargain, the Ct can determine the good intent of the parties and enforce the K.

4. Action in lawa. Black Industries, Inc. v. Bush

i. In order to declare a K, entered to w/o evidence of fraud, void against PP, must be found invalid on legal principles:

(a) D pays P to induce a public official to act in a certain manner;(b) K to do an illegal act;(c) K contemplating collusive bidding on a public K

ii. Profit earned was not as a result of either inducing gov’t action or interfering w/ competitive bidding – cannot be void as against PP here iii. The relative values of the consideration in a K between men dealing at arm’s length w/o fraud will not affect the validity of the K

(a) Ct does not believe that it should determine the validity of a K btw ordinary businessmen on the basis of its beliefs as to the adequacy of the consideration – would lead the Ct to having to determine the fairness of profits taken by all middlemen

5. Standard Form and Adhesion Ka. Doctrine of Reasonable Expectations – Whether a reasonable person in P’s position would have expected that the clause in question was present in the K.b. Boilerplates – Tends to be standard language that attorneys insert at the end of every agreement, w/o thinking much about it – govern the administration of the K

i. Ex.: choice of law, choice of forum, integration or merger, notice, fore majeure, waiverii. R2K § 211(3) – provides that where a party effectively manifests assent to a standardized expression of agreement, and the other party has reason to believe that he would not have done so if he had known that it contained a particular term, “the term is not part of the

Page 4: Contracts II – Outline

agreement”iii. Klar v. H&M Parcel – P left a package w/ room; stub had conspicuously printed the word “Contract” on the front –

(a) Ct held intention of the stub was to i.d. the package and not as a K(b) “In the mind of the bailor…piece of cardboard did not rise to the level of a K” “…D should be protected in its legal right to limit responsibility…public should also be safeguarded against imposition….If the bailee wishes to limit its liability for negligence, it must at least show that it has given adequate notice of the special K and

that is has received the assent…of those w/ whom it transacts business”(c) What was the intention of the transaction between the parties – Did P “expect” to be getting into a K – an acceptance is valid only if the offeree knows of the offer

c. Advantages & Disadvantages i. take advantage of lessons of experience; judicial interpretation of one helps to interpret

similar K; reduce uncertainty; makes risks calculableii. Adhesion K offer the means by which one party may impose its will upon another; no opportunity to bargain; take-it-or-leave-it proposition

d. O’Callaghan v. Waller & Beckwith Realty Co.i. P signed a lease (adhesion K) with an exculpatory clause and was injured from LL’s Negii. Judicial determinations of PP cannot readily take account of sporadic and transitory circumstances; should rest upon durable moral basis

(a) A lease clause exculpating a LL from liability for his own Neg should be upheld and is not void as against PP

iii. The relation of the lessor and lessee has been considered a private matter; the use of a form K does not of itself establish disparity of bargaining poweriv. Many Cts have made a distinction (in deciding whether or not to allow exculpatory clauses) between commercial and residential leases on grounds of the relative bargaining power of the parties

e. Graham v. Scissor-Tail, Inc. i. A K of adhesion is fully enforceable according to its terms, unless:

(a) A K or provision, which does not fall within the reasonable expectation of the weaker or “adhering” party will not be enforced against him(b) A principle of equity applicable to all K generally, a K or provision, even if consistent w/ the reasonable expectations of the parties will be denied enforcement if, considered in it context, it is unduly oppressive or “unconscionable”

6. Duty to Read/Duty to Disclosea. C/l rule: “in the absence of fraud, one who signs a written agreement is bound by its terms whether he read and understood it or not, or whether he can read or not”

i. Cts have relieved a party who has signed a standard form K that is unintelligibleii. Many states have “plain language ST”

b. Carnival Cruise Lines, Inc. v. Shutei. A forum selection clause is permissible if it is fundamentally fair – not unenforceable simply because it was not bargained for

(a) Assuming a risk on something not a necessity(b) Shutes were on notice that the forum selection clause was part of the agreement(c) [these] clauses limit the for a where cruise lines may be sued and save the time and expenses of litigating the forum issue – help reduce costs for cruise line – savings are passed on to the consumer(d) Important is type of goods being purchased – is this a necessity or a luxury item

7. Sources of Policing: Cts, Legislatures, and Agenciesa. Doe v. Great Expectations

ii. Where a consumer form K violates state law that provides for actual damages or a set amount, whichever is greater, a consumer is entitled to a full refund of the K price paid as

Page 5: Contracts II – Outline

restitutionary damagesiii. “…as a general rule judges are granted the discretion, under the proper circumstances, …to report instances of illegal conduct revealed in the course of proceeding before judge.”

(a) If shown a pattern of practice of breaking the law.iv. Cts police the use of standard form K not only by giving effect to protective legislation by determining whether the K adhere to statutory law, but also by reporting violations to various gov’t authorities, such as a state’s attorney general or consumer protection agency.

B. Unconscionability1. “Clauses involved are so one-sided as to be unconscionable” – 2-302

a. Is a meeting of the minds required for a K to be enforceable?i. No – subjective thought is not important; enforceability is based on an Objective standardii. Whatever terms or performance must meet the reasonable expectations of the parties

b. Principle is one of the prevention of oppression and unfair surprise and not of disturbance of allocation of risks because of superior bargaining power

i. Cts not likely to find unconscionability where both parties are sophisticated; a mere disparity of bargaining power is not enough

c. Whether a K is “unconscionable” is a question of law – i. UCC 2-302 – the issue of ‘unconscionability’ is not to be submitted to a jury; authorizes a Ct to refuse enforcement or to limit the application of a K or clause that it determines to have been ‘unconscionable’ ii. R2K § 208 – permits Cts to police for unconscionability

2. Elemental Approacha. Procedural Unconscionability – “fault” or “unfairness” due to unequal bargaining power – in the bargaining process

i. burdensome clauses in fine-print boilerplate; high-pressure salespeople who mislead the uneducated consumerii. However, when substantively unfair terms are present and are attributable to the bargaining dominance of the party favored by the terms…easy to conclude that the weaker party was deprived of free choice…amounted to procedural unfairness

b. Substantive Unconscionability– “fault” or “unfairness” in the bargaining outcome – unfairness of terms; unduly favorable to one of the parties

i. Cts are reluctant to get involved in re-writing a Kii. Value of consideration is not important iii. Price – Cts reluctant to find unconscionability w/o emphasizing procedural aspects of

unconscionabilityc. Separation is to make harmonious w/the bargain principle – ‘procedural’

i. Cts are starting to blur line between procedural and substantiveii. Sliding scale – if substantive unconscionability is great, less evidence of procedural is required (and vice versa)

d. Objections: i. When the doctrine of unconscionability is used in its substantive dimension, serves only to under cut the private right of contract; and the public policy of “freedom of K”ii. There will always be a disparity of bargaining power

3. Consumer Contracts (Cross-Collateralization clause)a. Williams v. Walker-Thomas Furniture Co.

i. Unconscionability has gen. been recognized to include an absence of meaningful choice on the part of one of the parties together w/ K terms which are unreasonably favorable to the other party

(a) Whether a meaningful choice is present in a particular case can only be determined by a consideration of all the circumstances surrounding the transaction(b) …meaningfulness of choice is negated by a gross inequality of bargaining power(c) …manner in which the K was entered into is also relevant

Page 6: Contracts II – Outline

iv. In determining reasonableness or fairness, the primary concern must be w/ the terms of the K considered in light of the circumstances existing when the K was made

4. Price Unconscionabilitya. Jones v. Star Credit Corp.

i. UCC 2-302 allows for not only a clause of the K, but the whole K to be found unconscionable as a matter of lawii. UCC 2-302 is not a simple mathematical formula, other factors:

(a) financial resources of the buyer known to the seller at the time of sale;(b) “knowing advantage” taken of the buyer; and (c) a gross inequality of bargaining power

iii. An example of substantive unconscionability – the Cts valuation of the goods is substituted for that of the parties

(a) Most Cts require some sort of procedural unconscionability5. Arbitration Clauses

a. Armendariz v. Foundation Health Psychcare Services, Inc.i. Issue is lack of mutuality – P had to arbitrate, but D did notii. It is unfairly one-sided for an employer w/ superior bargaining power to impose arbitration on the employee as P, but not accept such limitations when it seeks to prosecute a claim against the employeeiii. If an employer has reasonable justification for the arrangement – i.e., a justification grounded in something other than the employer’s desire to maximize its advantage based on the perceived superiority of the judicial forum – such an agreement would not be unconscionable

(a) …so long as the arbitration procedures permit the employees to vindicate their statutory rights

b. Scott v. Cingular Wireless i. An agreement that has a tendency to “be against the public good, or to be injurious to the public” violates public policy

(a) an agreement that violates PP may be deemed void and unenforceableii. “…the class action waiver before us is an unconscionable violation of this State’s policy to “protect the public and foster fair and honest competition”…it drastically forestalls attempts to vindicate consumer rights. To the extent that this clause prevents CPA cases, it is substantively unconscionable

c. Generally favored in law; however, where imposed in an adhesive K – must contain a “modicum of bilaterality” (not one-sided)

i. should not be cost-prohibitive; should not preclude class action; neutrality of arbiters; adequate discovery; limited means of judicial review

6. Remedies – both Restatement and UCC leave it to the discretion of the Ct to devise the most appropriate remedy

a. Refusal to enforce clause – strike out the offending clause, but enforce the rest of the Kb. Reformation – “reform” the offending clause (modify an “offending” price to make it reasonable)c. Refusal to enforce whole K – refuse to enforce the entire K, denying P any recovery at all

C. Good Faith1. Good faith is implied in all Ks under c/l and the UCC

a. A party is required to behave in a way that is consistent w/ the other party’s reasonable expectations about how the K will workb. Restatement § 205: Every K imposes upon each party a duty of good faith and fair dealing in performance and enforcementc. UCC § 1-201(19) – “Good faith” means honesty in fact in the conduct or transaction concernedd. UC § 2-103(1)(b) – “Good faith” in the case of a merchant means honesty in fact and the observance of reasonable commercial standards of fair dealing in the trade.

Page 7: Contracts II – Outline

e. Dalton v. Educational Testing Service i. Implicit in all Ks is a covenant of good faith and fair dealing

(a) Where the K is discretionary, this includes a pledge not to act arbitrarily or irrationally in exercising that discretion

ii. ETS failed to meet its obligation in not considering relevant material provided by Dalton and thus did not meet its covenant of good faith (required affirmative conduct)

f. Eastern Air Lines, Inc. v. Gulf Oil Corp.i. UCC § 2-306, specifically authorizes requirements Ks, provided that the requirements are not grossly disproportionate to stated estimates made in good faith

(a) all maters complained of are the fundamental given ingredients of the aviation fuel trade to which the parties have accommodated themselves (b) Cts look to:

(1) course of performance – performance during agreement(2) course of dealing – past dealing between the two parties(3) trade usage – how related businesses in the trade deal w/ each other

ii. The doctrine of “good faith” merely directs a Ct towards interpreting K within the commercial context in which they are created, performed and enforced, and does not create a separate duty of fairness and reasonableness which can be independently breached

g. Market Street Ass. v. Freyi. The duty of honesty, of good faith is not a duty of candorii. …to take deliberate advantage of an oversight by your K partner concerning his rights under the K is not the exploitation of superior knowledge or the avoidance of unbargained- for-expense; it is sharp-dealingiii. The essential issue is Ornstein’s state of mind…If O believed that Erb knew or would surely find out about paragraph 34, it was not dishonest or opportunistic to fail to flag that paragraph, or even to fail to mention the lease, in his correspondence and conversationsiv. “Opportunism” – K law does not require parties to be fair, or kind, or reasonable, or to share gains or losses equitably…It does not require parties to avoid taking advantage of the opportunities that arise from sequential performance, when the K does not cover a particular subj.

h. Bloor v. Falstaff Brewing Corp. i. A policy which emphasizes profit w/o fair consideration of the effect on sales volume will breach a “best efforts” clause to promote and maintain a high volume of sales of a certain product

(a) Damages based on a reasonably estimate of the royalties which would have been paid had such breach not occurred

ii. What are “best efforts”? Cts have looked to behavior of others engaged in like activitiesiii. “Best efforts” is a higher standard than “good faith;” requires reasonable efforts in light of ability and expectationsiv. UCC 2-306(2) – provides for the imposition of an obligation on the part of both parties to an agreement for exclusive dealing in goods to use their “best efforts.” It is a codification of the judicially made rule that was formed over the years, i.e., that certain Ks carried an implied obligation to use “best efforts” because they would otherwise lack mutuality of obligation.

2. Good Faith in Terminationa. Lockewill, Inc. v. United States Shoe Corp.

i. Suit for breach of exclusivity Kii. Where the parties to a franchise or exclusive agency…which is silent as to duration and which does not deal specifically w/ termination begin to perform…there agreement is construed to be terminable at the will of either party; limitations:

(a) …in any case of an indefinite agency where it is revoked by the principal, if it appears that the agent, induced by his appt., has in good faith incurred expense and devoted

Page 8: Contracts II – Outline

time and labor in the business w/o having had sufficient opportunity to recoup what expense he has incurred, the principal will be required to compensate him for such expense…the law will not permit one to deprive another of value w/o awarding just

compensation(b) The agent shall be afforded a reasonable opportunity to recoup the primary expenditures…if he has no such opportunity, the principal shall compensate him accordingly…the agent is entitled to compensation on a quantum meruit basis rather than ordinary damages for breach of K

(1) Cts are reluctant to interpret K providing unlimited contractual right unless the K clearly states that that is the intention of the parties

3. Satisfaction Conditionsa. Where a K gives one party absolute discretion as to whether the K terms were performed to the party’s satisfaction:

i. Commercial quality, operative fitness or mechanical utility – objective standardii. Aesthetics or fancy – subjective standard

b. R2K § 228:i. “When…it is practicable to determine whether a reasonable person in the position of the obligor would be satisfied, an interpretation is preferred under which the condition occurs if such a reasonable person in the position of the obligor would be satisfied.”ii. Greater tolerance for subj. test where condition is satisfaction of an independent 3d party

D. Public Policy – Illegal Ks1. Statutory Violations

a. Bovard v. American Horse Enterprises, Inc.ii. Cts will not enforce Ks which violate public policy considerations…furthermore, the Ct will not permit the parties to maintain an action to settle or compromise a claim based on an illegal Kiii. PP involves a degree of subjectivity; unless it is entirely plain that a K is violative of sound PP, a Ct will never declare soiv. Burden is on D to show enforcement of the K would be a violation of settle PPv. Factors to consider in analyzing whether a K violates PP:

(a) Nature of the conduct; extent of public harm that may be involved; the moral quality of the conduct of the parties in light of the prevailing standards of the community

b. In Pari Delictoi. In circumstances of equal fault, the position of the D is the more compelling. ii. “While it may not always seem an honorable thing to do, a party to an illegal agreement is permitted to set up the illegality as a defense even though the party may be alleging his or her own fault.”iii. In some exceptional cases, a party in pari delicto may obtain restitutionary reliefiv. Rest.: Restitution is available to a party “who would otherwise suffer a forfeiture that is disproportionate in relation to the contravention of PP involved.”

2. Inducing Official Actiona. “The authorities very generally hold that a K to pay for services to be performed in the endeavor to obtain or defeat legislators, such as, for the exertion of personal political influence apart from the appeal to reason as applied to the consideration of the merits or demerits of the legislation in question, in an illegal K”

3. Briberya. X.L.O. Concrete Corp. v. Rivergate Corp.

i. Antitrust defenses to a K will be upheld where enforcement of the K would compel the precise conduct made unlawful by the antitrust laws.ii. A K which is legal on its face and does not call for unlawful conduct in its performance is not voidable simply because it resulted from an antitrust conspiracyiii. The critical question is whether the K is so integrally related to the agreement,

Page 9: Contracts II – Outline

arrangement or combination in restraint of competition that its enforcement would result in compelling performance of the precise conduct made unlawful by the antitrust laws

(a) also, whether the K price was excessive, and whether sustaining the illegality defense would render the K void

iv. Cts should avoid upholding antitrust defenses in K cases where doing so would work a substantial forfeiture on one party while unjustly enriching the otherv. The relative culpability, bargaining power, and knowledge of the parties to the K should also be considered in assessing the possibility of unjust enrichment.vi. Cts may be swayed if D had knowledge of the improper act and appears to be getting something for nothing

b. The bribe itself is unenforceable as a promise inducing violation of an agent’s fiduciary duty4. Licensing

a. When the purpose of a licensing requirement is ascertained to be raising public revenue, and not the protection of the public’s welfare – health, morals or the like – a claimant’s want of a license is generally not a bar.

E. Judicially Created Public Policy1. Restraints of Trade

a. Hopper v. All Pet Animal Clinici. A covenant not to compete is valid only if the restraint is no greater than is required for the protection of the employer, does not impose undue hardship on the employee, and is not injurious to the public.ii. Covenants to compete are construed against the party seeking to enforce them…the initial burden is on the employer to prove the covenant is reasonable and has a fair relation to, and is necessary for, the business interests for which protection is sought.iii. 2 PP in conflict: Freedom to work & Freedom to Kiv. Enforceability of a cov. Not to compete – proper balance exists between the competing interests of the employer and employeev. The reasonableness of the limitations placed on a former employee by a covenant not to compete are determinations make by the Ct as a matter of law

b. Cts used to either enforce the clause as written or reject it all together; the trend is now toward judicial modification:

i. “Blue Pencil” Rule: Ct crosses out, or “blue pencil” words “to the extent that a grammatically meaningful reasonable restriction remains after the words making the restriction unreasonable are stricken.”ii. Rule of Reasonableness: the cov. Will be enforced only to the extent reasonably necessary to protect the employer’s interest – R2K § 184 follows this approach

2. Termination of Ks – At-will Employeesa. Sheets v. Teddy’s Fronted Foods

i. An employee hired at-will may maintain an action for wrongful discharge if he can prove that the discharge violates PP

(a) An employer cannot, w/o liability, terminate an employee for exercising some right available to him at law or from refraining to do something in violation of the law; an employee should not have to choose btw a living and illegal conduct

ii. Cts generally have been unwilling to hold that an employer is under a duty of god faith and fair dealing that allows discharge only for a good causeiii. Cts will interfere only if it implicates an explicit ST or Const. provision, or judicially conceived notion of PP

b. Balla v. Gambro, Inc.i. The PP of protecting lives and property of citizens is adequately safeguarded w/o extending the tort of retaliatory discharge to an in-house counselii. An in-house counsel does not have a choice of whether to follow their ethical obligations as attorneys or follow the illegal and unethical demands of their clients – they must abide

Page 10: Contracts II – Outline

by the Rules of Professional Conductiii. Employers might be less willing to be forthright and candid w/ their in-house counsel for advice regarding potentially questionable corporate conduct, knowing that their in-house counsel could use this info. In a retaliatory charge.

3. Family Relationsa. Simeone v. Simeone

i. Contracting parties are normally bound by their agreements, w/o regard to whether the terms of thereof were read and fully understood and irrespective of whether the agreement embodied reasonable or good bargains

(a) Prenups are Ks and should be treated as suchii. If parties viewed an agreement as reasonable at the time of its inception, as evidenced by their having signed the agreement, they should be foreclosed from later trying to evade its terms by asserting that it was not in fact reasonableiii. Shift from paternalism to equal rights

b. In the Matter of Baby Mi. Surrogate Ks violate PP and are unenforceable

III. Remedies for BreachA. Equitable Remedies

1. Specific Performance – orders the promisor to render the promised performancea. Campbell Soup Co. v. Wentz

i. A party may have SP of a K for the sale of chattels if the legal remedy is inadequate. Inadequacy of the legal remedy is necessarily a matter to be determined by an examination of the facts in each particular instanceii. The test for SP is not necessarily “objective” – equity has given it to enforce K for articles – family heirlooms, etc. – the value of which was personal to Piii. When a purchaser has contracted in advance for goods of a special type that are unavailable on the open market and that support the purchaser’s good reputation, SP should be granted where the seller has breached the K

(a) Legal remedies would be inadequate: virtually impossible to obtain the specific carrots on the open market, were of a specific type, and had been contracted for in advance in anticipation of Campbell’s needs

iv. Equitable relief was ultimately denied due to the K being to one-sided for Campbellv. Uncertainty – Campbell can’t tell what it’s damages are – SP allowed

b. Klein v. PepsiCo, Inc. i. If the goods are unique, a jilted buyer of goods may seek SPii. SP is inappropriate where damages are recoverable and adequate

(a) An increase in the cost of a replacement does not merit the remedy of SPiii. Under the UCC, Klein had two alternative remedies:

(a) UCC § 2-713 – it is assumed he could have mitigated his loss by purchasing a substitute aircraft; his damages would have been the difference between the market price and the K price(b) Had he actually “covered” by purchasing another GII, his damages would have been the difference btw the cost of “cover” and the K price

c. Morris v. Sparrowi. An action for SP for the delivery of personal property may be maintained in equity where the personal property is unique – Sparrow spent a lot of time training the horseii. Often when performance might provide a peculiar and unique value, Cts express the view that money damages are too speculative or uncertain, but in essence they are ruling that it might be too difficult, or a drain on Ct resources, to make such a determination

d. Specific Performance and Investmenti. Efficient Breach: When a breaching promisor can fully compensate the promisee, breach may be Pareto efficient: the promisee can be left no worse off and the promisor off by the

Page 11: Contracts II – Outline

breach.(a) SP can encourage this kind of possibility of breach, leaving parties to invest as though they are certain to receive performance: either the promisor will perform or the Ct will order SP

ii. An order of SP does not necessarily imply that parties will actually perform – they commonly reach private settlements, even after receiving a Ct order

e. Laclede Gas Co. v. Amoco Oil Co.i. Specific performance is available as a remedy for breach of a long-term supply K.

(a) hard to replace w/ another long term K due to economic conditions(b) there would be a lot of transaction costs w/ all the short term Ks to fill the job

ii. R2K § 370 – SP will not be decreed unless the terms of the K are so expressed that the Ct can determine w/ reasonable certainty what is the duty of each party and the conditions under which performance is due

f. Northern Delaware Industrial Development Corp. v. E.W. Bliss Co.i. Equity should not order SP of any building K in a situation in which it would be impractical to carry out such an order…unless there are special circumstances or the public interest is directly involvedii. Performance of a K for personal services, will not be enforced

g. Walgreen Co. v. Sara Creek Property Co. i. When the costs of injunctive relief are less than the costs of a damages remedy, injunctive relief is an appropriate remedy, even when the damages remedy is not shown to be inadequateii. Damages remedy would have been difficult to compute – calculation of lost profits for at least 10 years into the futureiii. Injunction –

(a) removes evidentiary evidence form the Ct; (b) supervised by Walgreen, so minimal costs for the Ct, and becomes a private matter between the parties(c) price and costs are more accurately determined by the market rather than gov’t; battle of experts is a less reliable method

h. Limitationsi. Inadequacy of money relief

(a) grounded in the belief that money offers no substitute for the promised performance(b) too speculative – when performance would provide a “peculiar and unique value”

ii. Definiteness: Ct will not give equitable relief unless the K’s terms are definite enough to enable the Ct to frame an adequate order

iii. Difficulty to enforce: If there are likely to be significant difficulties in enforcing and supervising the order, Ct will not order

i. Personal Services: A Ct will not order SP of a K to provide a service that is personal in nature

i. Difficulty is passing judgment on quality of performanceii. The undesirability of compelling the continuance of personal relations after

disputes have arisen, and imposing what might seem like involuntary servitude)iii. Injunction: Ct is willing to grant an injunction

j. Sale of Goods:i. SP is likely in cases of output an requirements K

ii. If goods are scarce, doesn’t mean it’s unattainable – just have to pay a higher price so damages would be higher

2. Injunction and other Equitable remediesa. Injunction: directs a party to refrain form doing a particular act.

Page 12: Contracts II – Outline

i. Cts are willing to enjoin parties from other performances in appropriate circumstancesii. Cts will not grant an injunction unless the remedy in damages would be inadequate

b. Replevin – buyer’s right—UCC § 2-716(3) – assuming unavailability of “cover”c. Action for Price – seller’s right –UCC § 2-709 – assuming unable to resell (for a buyer to pay for goods)

B. Measuring Damages1. Expectation: put the plaintiff in the position he would have been in had the K been performed;

“benefit of the bargain” (including any profits)a. R2K § 347: Right to damages based on expectation interest as measured by – (a) loss in value + (b) other loss – (c) cost and loss avoided

(a) loss in value (difference in value of what should have received and the value of what was received) of the other party’s performance caused by its failure or deficiency plus(b) any other loss, including incidental or consequential loss, caused by the breach, less

(1) Incidental losses – cost incurred in a reasonable effort, whether successful or not, to avoid loss –fees you have to pay, transportation costs

(2) Consequential damages - damages that wouldn’t normally flow from the breach in question; due to the unique circumstances of the P; can recover if they were foreseeable by D @ the time the K was formed or in reasonable contemplation of both contracting parties.

(c) any cost or other loss that [he or she] has avoided by not having to perform(1) a beneficial effect on the injured party by saving that party further expense that would have been incurred had the performance continued (2) effect allowing that party to avoid some loss by salvaging and reallocating some or all of the resources that it otherwise would have had to devote to performance of the K (3) Does not include any “overhead”

b. Damages (E&E)= Gross Profits (K price – total direct cost) + reliance expenditure – (any) payments receivedc. P must prove their existence and amount to a “reasonable certainty”

2. Reliance: puts the plaintiff in the position he would have been in had the K never been made; the amount the plaintiff spent in performing or in preparing to perform; changing position in response to the promise; awarded when:

a. Profits too speculative: expectation damages cannot be computed because P’s lost profits are too speculative or uncertainb. Vendee in Land K: P is buyer and seller refuses to perform (down payment plus any reasonable expenses)c. Promissory Estoppel: quasi-K theory; less than expectation damages (suit is not on the K) and more than nothing (prevent an injustice)

3. Restitution: the value to the defendant of the plaintiff’s performance. Goal is to prevent unjust enrichment.

a. A non-breaching P who has partly performed before the other paty breached may bring suit on the K, and not be limited by the K priceb. A breaching P who has not substantially performed may bring a quasi-K suit and rcover the value that she has conferred upon the Dc. Not limited by the K price

i. Not available where P has fully performed4. Under UCC – Article 2

a. Reserves the remedy for specific performance for special cases; treats damages as the primary remedy for breach

i. § 2-709 allows seller to claim the price of the goods only when the goods have been ccepted by the buyer or they are incapable of being resold (lost or damaged)ii. § 2-716 allows buyer to claim specific performance when the goods are unique

Page 13: Contracts II – Outline

b. § 2-713 – Buyer’s Damages for Nondelivery or Repudiationi. …If the seller wrongfully fails to deliver or repudiates [goods]

(a) the measure of damages…is the difference between the market price at the time for tender…and the K price together with any incidental or consequential damages…but

less expenses saved in consequence of the seller’s breachc. § 2-712 – “Cover”; Buyer’s Procurement of Substitute Goods

i. If the seller wrongfully fails to deliver or repudiates…the buyer may “cover” by making in good faith and w/o unreasonable delay any reasonable purchase of or K to purchase goods

in substitution for those due from the sellerd. § 2-708(1) – damages for seller based on a hypothetical resale – difference between the K price and the market price of the gods at the time and place at which delivery was to have been tendered under the Ke. § 2-708(2) – Calculating the seller’s lost profit:

i. Recovery = Gross profit (K price – total direct cost) + any reliance expenditures – (any payments received or salvage)

f. § 2-706 – permits an aggrieved seller to enter a substitute transaction by reselling the goods and, provided that the resale is made in good faith and in a commercially reasonable manner, to

recover the shortfall between the K price and the resale price 5. Lost Profits and Fixed Costs

a. Vitex Manufacturing Corp. v. Caribtex Corp.i. Should overhead be treated as part of the gross profits and recoverable at breach, or considered as part of the seller’s costs?ii. In a claim for lost profits, overhead should be treated as part of the gross profits and recoverable as damages, and should not be considered as part of the seller’s costs.iii. Reliance damages from relying on the K by reopening its plant, recalling employees, and taking other necessary steps to begin processing.

b. Laredo Hides v. H&H Meat Productsi. Laredo “covered” when H&H failed to deliver hides under the K.ii. When a seller wrongfully repudiates a K or fails to make delivery of the goods thereunder, the buyer may “cover” by obtaining such goods elsewhere and sue the seller for the

difference between the cost of cover and the K price plus any incidental or consequential damages. UCC § 2-712

(a) Seller has the burden of proving that “cover” was not properly obtainediii. Buyer may also recover damages measured by the difference between the market price at the time when the buyer learned of the breach and the K price together with any incidental and consequential damages.

(a) Market price can be higher or lower – harder to figure out; “cover” (if reasonable) is easy

c. Substitute Transactions: Servicesi. If an employee is fired in breach of a K and does other work as a result of being fired from the K, the employee’s damages are based on the salary that would have been earned under the broken K less that earned by doing the other workii. “Collateral Source Rule” – an employee’s recovery in tort is reduced by sums received from a collateral source, such as unemployment compensation and other benefits, in order to avoid double recoveryiii. “Alternative Product” Rule – a seller, in breach, contests a buyer’s claim of loss by

contending that, aside from the K in question, nothing would have averted the loss(a) a recovery of a claimant’s lost expectation cannot be justified if, when the K was made, that party could not have accomplished its purpose by dealing with someone else, and so lost no opportunity.

d. R.E. Davis Chemical Corp. v. Diasonics, Inc.i. D claimed it lost a “volume sale” when P breached a K of sale

Page 14: Contracts II – Outline

ii. If the seller would have made the sale represented by the resale whether or not the breach occurred, damages measured by the difference between the K price and market price cannot put the lost volume seller in as good a position as it would have been in had the buyer performed. iii. The breach effectively cost the seller a “profit” …seller can only be made whole by awarding it damages in the amount of its “lost profit” from the original sale if he can show that the subsequent sale would have occurred absent the breach and would have been profitable.

e. Lost Volume Selleri. A seller who can accommodate more than one buyer and for whom a buyer’s breach does not release the goods for sale to another customer; in such a case, the appropriate measure of damages is the net profit the seller would have earned pursuant to the sale.ii. “…the relevant questions include, not only whether the seller could have produced the breached units in addition to its actual volume, but also whether it would have been profitable for the seller to produce both units…”iii. B/O/P is on the seller to prove that they can produce another one & sell it @ a profit

f. United States v. Algernon Blair, Inc. i. A promisee is allowed to recover in quantum meruit the value of services he gave to a

defendant who breached the K irrespective of whether he would have lost money had the K been fully performed and would thus be precluded from recovering in a suit on the K.ii. The standard for measuring the reasonable value of the services rendered is the amount for which such services could have been purchased from one in the plaintiff’s position at the time and place the services were renderediii. [in construction Ks] a promisee upon breach has the option to forgo any suit on the K and claim only the reasonable value of his performance.

(a) No right to restitution when K is fully performed(b) Problem when K is mostly completed, then there is a breach & party can choose between K price & restitution damages

C. Limitations on Damages1. Avoidability

a. Rest. § 350:i. …damages are not recoverable for loss that the injured party could have avoided w/o undue

risk, burden or humiliationb. UCC 2-713:

i. Measure of damages is difference in market price and contract pricec. The injured party is under a “duty” to mitigate damages…the injured party incurs no liability to the party in breach for a failure to mitigate.d. Rockingham County v. Luten Bridge Co.

i. After an absolute repudiation or refusal to perform by one party to a K, the other party cannot continue to perform and recover damages based on full performance

(a) P cannot hold D liable for damages, which need not have been incurred…ii. Under UCC 2-704(2), a seller that is to manufacture goods may proceed to complete their manufacture upon the buyer’s repudiation… “in the exercise of reasonable commercial judgment for the purposes of avoiding loss and of effective realization.”

(a) The seller that does so may then base recovery on the goods as completed, even if the “reasonable commercial judgment” turned out to be wrong

2. Avoidability Under Contracts For the Sale of Goodsa. Tongish v. Thomas

i. Tongish breached; Thomas wanted market price measure of damages instead of actual loss;ii. When seller breaches, market damages should be awarded even though in excess of the buyer’s actual loss

Page 15: Contracts II – Outline

iii. Market Damages Rule – (majority view) –UCC 2-713 – difference between the market price at the time the buyer learned of the breach and the K priceiv. Lost Profits Rule – (minority view) – reduce market damages to the P’s loss w/o regard to whether this creates a windfall for the non-breaching party

b. Parker v. Twentieth Century – Fox Film Corp.i. The general measure of recovery by a wrong-fully discharged employee is the amount of salary agreed upon for the period of service, less the amount which the employer affirmatively proves the employee has earned or with reasonable effort might have earned from other employmentii. The employee’s rejection of or failure to seek other available employment of a different or inferior kind may not be used to mitigate damages

(a) The person with a duty to mitigate need not expose himself to undue risk, expense, or humiliation; in mitigating, the employee also does not have to accept a position of lesser rank or at a lower salary.

iii. Damages for injury to the reputation of the employee are considered to be too remote 3. Avoidability and the Cost of Remedy Defect

a. Jacob & Youngs v. Kenti. An omission, both trivial and innocent, will sometimes be atoned for by allowance of the remitting damage and will not always be the breach of a condition to be followed by forfeiture.ii. It is true that in most construction cases the cost of replacement is the measure; the owner is entitled to the money, which will permit him to complete, unless the cost of completion is grossly and unfairly out of proportion to the good to be attained. When it is true, the measure is the difference in value. – The Value Ruleiii. Substantial performance cannot occur where the breach is intentional, as it is the antithesis of material breach.

b. Groves v. John Wunder Co. i. Value of the land as distinguished from the value of the intended product of the K, which ordinarily will be equivalent to its reasonable cost, is no proper part of any measure of damages for willful breach of a building K -

(a) Never before …the lack of value in the land furnished to the contractor who had bound himself to improve it, gave any escape from the ordinary consequences of a breach of K(b) Even in the case of substantial performance in good faith…it is error to instruct the measure of damage is “the difference in value between the house as it was and as it would have been if constructed according to the K”

ii. The “correct doctrine” is that the cost of remedying the defect is the “proper” measure of damages – Cost of Performance Ruleiii. To diminish damages recoverable against him in proportion as there is presently small value in the land would favor the faithless contractor & ignore P’s right to Kiv. The injury is what was promised to be done to alter the landv. There can be no unconscionable enrichment…when the result is but to give one party to a K only what the other has promised…particularly where the delinquent has had full payment for the promised performancevi. To allow the value rule in this situation, a land owner with land low in value, would place the land owner at the mercy of unethical contractorsvii. Economic Waste will keep a Ct from approving a large amount of damages (tearing out one manufacture of plumbing for the same quality but with a different brand) to correct a poorly performed K (awards difference in value); unless if it is shown that the owner’s purpose was to satisfy some personal taste or fancy, damages equal to the cost of completion will be awarded even if is involves economic waste.

c. Peevyhouse v. Garland Coal & Mining Co.

Page 16: Contracts II – Outline

i. “Cost of Performance Rule” v. “Value Rule”(a) “…In cases where the defect is one that can be repaired or cured without undue expense…the cost of performance is the proper measure of damages, but where the defect in material or construction is one that cannot be remedied w/o an expenditure for reconstruction disproportionate to the end to be attained, the value rule should be

followed.”ii. Where a contract provision breached is merely incidental to the main purpose, and where the economic benefit which would result to lessor by full performance of the work is

grossly disproportionate to the cost of performance, the damages which lessor may recover are limited to the diminution in value resulting to the premises because of the nonperformance.

4. Foreseeabilitya. Hadley v. Baxendale

i. The damages…should be such as may fairly and reasonably be considered either arising naturally, i.e., according to the usual course of things, from such breach of K itself, or such as may reasonably be supposed to have been in the contemplation of both parties, at the time they made the K, as the probable result of the breach of it – iii. If the special circumstances were communicated to the D’s, damages resulting from the breach would be the amount of injury under these special circumstancesiv. If the special circumstances were unknown, damages from the breach would arise generally and w/o regard to the special circumstancesv. Risk of loss wasn’t on breaching party; damages weren’t foreseeable; danger need not only be foreseeable but the breaching party must know and agree

b. Foreseeability and Assumption of Risk are ways of describing the bargain; the loss must be foreseeable when the K is entered into; it cannot be overly speculative

i. If a seller knows it is selling to a buyer for resale, loss of such profits is generally regarded as foreseeable. Loss of profits on future Ks is another matterii. R2K § 351(3) – confronts the problem of risk by stating that a Ct may limit damages even for foreseeable loss “if it concludes that in the circumstances justice so requires in order to avoid disproportionate compensation.

c. Delchi Carrier Spa v. Rotorex Corp . i. Damages for breach of K…may not exceed the loss which the party in breach foresaw or ought to have foreseen at the time of the conclusion of the K, in the light of the facts and matters of which he then knew or ought to have known, as a possible consequence of the breach of contract.ii. Lost profits on a resale are consequential damages; UCC permits them for buyers but not for sellersiii. A seller who knows a buyer is going to be reselling is going to be liable for consequential damages from lost profits if foreseeable – future Ks are too speculative

d. UCC 2-715(2) Consequential Damages – available to buyeri. (a) any loss resulting from general or particular requirements and needs of which the seller at the time of contracting had reason to know and which could not reasonably be prevented by cover or otherwise; and ii. (b) injury to person or property proximately resulting from any breach of warranty

e. Kenford Co. v. County of Eriei. In order to impose extraordinary damages on a breaching party, such damages must have been brought within the contemplation of the parties as the probable result of a breach at or prior to the time of contracting.ii. In determining the reasonable contemplation of the parties, the nature, purpose and particular circumstances of the K known by the parties should be considered as well as “what liability the D fairly may be supposed to have assumed consciously, or to have warranted the P reasonably to suppose that it assumed, when the K was made

Page 17: Contracts II – Outline

(a) Damages may have been foreseeable, as the County knew he was buying up the land in the periphery of the proposed stadium. However, the damages were too speculative.(b) Buying up the land was not a requirement of the K

f. Restatement § 351i. Damages are not recoverable for loss that the party in breach did not have reason to foresee as a probable result of the breach when the K was madeii. Loss may be foreseeable…because it follows from the breach

(a) in the ordinary course of events, or(b) as a result of special circumstances, beyond the ordinary course of events, that the party in breach had reason to know

iii. A court may limit damages for foreseeable loss by excluding recovery for loss of profits, by allowing recovery only for loss incurred in reliance, or otherwise if it concludes that in the circumstances justice so requires in order to avoid disproportionate compensation

g. Sentimental Valuei. Damages may be foreseeable. However, they are too speculative for a Ct to award

h. Emotional Distressi. Cts are reluctant to allow damages for EDii. Sometimes ED is not foreseeable, difficult to measure; Cts have likened to punitive

damages; Cts may be more likely to award punitive damages than for EDiii. Punitive Damages “should be awarded only in the most unusual and compelling circumstances” and will be sustained on appeal only when it is shown that the D acted in a manner that was an extreme deviation form reasonable standards of conduct…the act was performed by the D with an understanding of or disregard for its likely consequences

5. Certaintya. Generally

i. Damages for a breach of K must “be shown, by clear and satisfactory evidence, to have been actually sustained” and “be shown with certainty, and not left to speculation or conjecture.”

(a) contemporary formulations insist on ‘reasonable certainty’(b) R2K § 352 – precludes recovery “for loss beyond an amount that the evidence permits to be established w/ reasonable certainty”(c) Comment 1 to UCC 1-106 – explains that damages need not “be calculable w/ mathematical accuracy,” are “at best approximate,” and “have to be proved w/ whatever definiteness and accuracy the facts permit, but no more.”(d) The injured party has a more onerous burden that “preponderance of evidence”

ii. “Lost Reputation” – Am Cts generally don’t award, to hard to calculate; or unforeseeableiii. Cts will not award damages for sentimental value; value of chance (Cts are split); emotional distress; artistic creations…(Loss of “Good Will” (corporations) YES)

b. Fera v. Village Plaza, Inc. i. How to calculate profits for a new business?ii. If a business is one that has already been established a reasonable prediction can often be made as to its future on the basis of its past history; if a business has not had such a history as to make it possible to prove w/ reasonable accuracy what its profits have been in fact, the profits prevented are often but not necessarily too uncertain for recovery.iii. Future profits as an element of damage are in no case excluded merely because they are profits but because they are uncertain. When they are established w/ reasonable certainty, they are allowed

(a) Here, there was enough evidence to where the jury didn’t have to guess.D. “Liquidated Damages” and “Penalties”

1. Summarya. Law will not award penalties; sometimes a promisee can encourage the promisor’s timely performance by stipulating liquidated damages…the promisor must agree to the stipulated

Page 18: Contracts II – Outline

damages and will likely demand compensation for being accountable for an award greater than the legal defaultb. Promisors should be expected to demand compensation for facing a risk of paying high liquidated damages in the event of a breachc. Wasserman’s Inc. v. Township of Middletown

i. Liquidated damages clauses are unreasonable and not enforced if it does more than compensate the Plaintiffs for their approximate damages caused by the breachii. A sub-issue is when the reasonableness of a damages provision is challenged is that of timing:

(a) Single Look – considering LD clause at the time the K is made (if reasonable)(b) Second Look – to assess reasonableness at either the time the K was made or at

the time of the breach (modern trend is to assess at either time)(increases chance of litigation)

(c) UCC reflects this trend in 2-718 w/ a reference to “anticipated (when K was formed) or actual harm (when K was breached)”

d. Summary Con’t. i. Arguments for: Cts don’t have to waste time listening to experts; Parties can control exposure to risk; Freedom of K; efficientii. Contractual clause providing the amount of damages in the case of a K breach; valid, if:

(a) Clause must reasonably forecast the probable loss due to breach (as calculated at K formation)(b) Harm caused by the breach must be difficult to calculate, and (c) Parties must tailor the clause to the K’s circumstances (i.e., it cannot be a penalty)(d) CiDeR – Circumstances; Difficulty of Calculation; Reasonable forecast

d. Dave Gustafson & Co v. State i. “A provision for payment of a stipulated sum as a liquidation of damages will ordinarily be sustained if it appears that at the time the K was made the damages in the event of breach will be incapable or very difficult of accurate estimation, that there was a reasonable endeavor by the parties as stated to fix fair compensation, and that the amount stipulated bears a reasonable relation to probable damages and not disproportionate to any damages reasonably to be anticipated.”ii. When they are fair and reasonable attempts to fix just compensation for anticipated loss caused by breach of K…enforced.

IV. Performance and Breach: A. Conditions

1. Express Conditionsa. Permits a party to defer or withhold its own performance in the face of a threat that it will not receive what was promised in exchange (conditions and right to demand assurance)

i. Something that has to happen before a duty can be triggeredii. Substantial Compliance is not sufficient strict complianceiii. If condition is dependent on satisfaction of one party, “good faith” is required. If not purely a matter of taste, Ct will apply a “commercially reasonable standard”

b. Luttinger v. Roseni. A condition precedent is a fact or event, which the parties intend must exit or take place before there is a right to performance; if the condition precedent is not fulfilled, the K is not enforceable

(a) Here, K’s language was unambiguous and clearly indicated the condition precedent to take place before P was to purchase D’s premises

(b) Entitled to refund of depositii. Cts will enforce to the letter because it was an express condition: Express conditions require strict compliance

Page 19: Contracts II – Outline

c. Internatio-Rotterdam, Inc. v. River Brand Rice Mills, Inc.i. A condition, which is an act or event, other than a lapse of time, must be literally complied with.ii. The non-occurrence of that condition entitled D to rescind or to treat its contractual obligations as discharged.iii. Buyer (Internatio) took the risk that the price would go down; seller (River) took the risk that the price would go up; however, after 17th, Seller took the risk both ways; that why the condition of notice of where to ship the rice was important;

(a) the condition of timely notice was expressediv. The importance of prompt delivery by a seller of goods generally derives form the circumstances that goods, as contrasted w/ land, are particularly likely to be subject to rapid fluctuations in the market

2. Problems of Interpretation: Condition, Duty, or Botha. Peacock Construction Co. v. Modern Air Conditioning, Inc.

i. Ambiguous provisions in subcontracts which do not expressly shift (in the contract) payment failure by the owner to the subcontractor, will be interpreted as constituting absolute promise to pay, and not as payment received by the owner as a condition precedentii. If a condition exists is a question of law; interpretation of a document is a question of law; iii. If an issue of K interpretation concerns the intention of the parties, that intention may be determined form the written K, as a matter of law, when the nature of the transaction lends itself to judicial interpretation

b. Gibson v. Cranagei. Where two parties agree that one of them must be “personally satisfied” before liability will arise, that party may insist on his right to personal satisfactionii. A minimum “good faith” requirement must be imposed; and no violation of PPiii. If satisfaction is for a 3rd person (reselling) – a “commercially reasonable” standard

3. Problems of Interpretation: Third-Party Satisfactiona. Some of the risk inherent in making the other party’s duty conditional on its own satisfaction can be eliminated by making it duty conditional instead on the satisfaction of an independent 3d party, perhaps an expert of some kind.b. If a 3rd party’s judgment is egregiously bad, Ct will try not to enforce condition

4. Mitigating Doctrines: Prevention, Waiver, Estoppel, and Election (express conditions)a. Prevention: - One who prevents the occurrence of a condition of one’s own duty may be precluded from later asserting the non-occurrence of that condition; “good faith” requires one do nothing to prevent the occurrence of such a conditionb. Waiver: consideration is not necessary; an obligor whose duty is conditional may promise to perform despite the nonoccurrence or delay of the condition

i. Rest. § 84 – the excuse of the nonoccurrence of a condition of a duty – not a relinquishment of a known right and the termination of the reciprocal duty

ii. Waiver after the time of occurrence of the condition cannot be retractediii. Cannot waive a dutyiv. If ambiguous, Ct will find a duty. Breach of a duty pay damages; breach a condition other party is off the hook, K is forfeited: Cts abhor forfeiture

c. Estoppel:i. A party that, w/o consideration, has waived a condition that is within the other party’s

control before the time for occurrence of the condition can retract the waiver and reinstate the requirement that the condition occur unless the other party has relied to such an extent that retraction would be unjust – estoppel will preclude the retraction

d. Election:i. A choice…a binding one on the party who makes itii. When the time for occurrence of a condition has expired, the party whose duty is conditional has a choice between taking advantage of the nonoccurrence of the condition

Page 20: Contracts II – Outline

and treating the duty as discharged or, disregarding the nonoccurrence of the condition and treating the duty as unconditional

e. McKenna v. Vernoni. “If he waived it repeatedly, as he did here, during the progress of the work, he cannot complain if he be held to have waived it when he seeks to defend against a final payment for work done….especially since the work was done under his own observation w/o

complaint.”6. Mitigating Doctrines: Interpretation and Avoidance of Forfeiture (express conditions)

a. Cts have traditionally preferred interpretations of K language that avoid forfeiture.i. Language making a promise conditional may be interpreted in such a way that the condition has occurredii. As where a condition of satisfaction is taken to be reasonable satisfactioniii. Cts will read as a duty whether than a condition

b. Hicks v. Bushi. Evidence of oral conditions is not to be excluded as contradictory or “inconsistent” merely because the written agreement contains other conditions precedent…

(a) PER will not bar proof of an orally established condition precedent, but only those that contradict the terms of the written agreement

B. Constructive Conditions of Exchange1. Constructive Conditions

a. Kingston v. Prestoni. Mutual and Independent Covenants: either party may recover damages from the other for the injury he may have received by a breach of the covenants in his favor, and where it is no

excuse for the defendant to allege a breach of the covenants on the part of the plaintiffii. Conditions and Dependent Covenants: performance of one depends on the prior performance of another, and therefore, till this prior condition is performed, the other party is not liable to an action on his covenant iii. Mutual Covenants to be performed at the same time: if one party was ready and offered to perform his part, and the other neglected or refused to perform his, he who was ready and offered has fulfilled his engagement, and may maintain an action for the default of the other.iv. The dependence or independence of covenants was to be collected from the evident sense and meanings of the parties, which in turn will normally be determined by the “order of time in which the intent of the transaction requires their performance.”

b. Duties of each party are dependent on each other. One is an implied condition on the otheri. Substantial Compliance is sufficient

c. Default rule is that Cts will read into a K a constructive condition of exchange; need to show they are independent of each other

i. Constructive conditions giver the person who performs last an advantage2. Time for Performance

a. Because of the doctrine of constructive conditions, fixing the time for performance under a K has the important effect of allocating the risk that one party will perform but will not receive the other party’s return performanceb. Party’s can fix the times for performance in their K; if the K is silent, default rules may applyc. Stewart v. Newbury

i. Where the K is made to perform work and no agreement is made as to payment, the work must be substantially performed before payment can be demanded

(a) progress payments are not read into the K; periodic payments are not implied(b) If periodic payments are agreed to, performance is a constructive condition precedent to first payment which is a constructive condition precedent to the next stage of performance

Page 21: Contracts II – Outline

C. Mitigating Doctrines (constructive conditions)1. Substantial Performance

a. Substantial performance is often the doctrine where constructive rather than express conditions are concerned; does not materially breach

i. Substantial performance is a question of factii. Factors important: amount of work done, what the forfeiture would be, availability of compensation, good faith, unjust enrichment, is it a material breach?

b. When one party’s performance is substantial, the other party is not entitled to withhold any return performance that is due but has the right to claim damages for breach

c. Jacob & Youngs v. Kenti. Substantial performance cannot occur where the breach is intentional, as it is the antithesis of material breach. The part unperformed must not destroy the purpose or value of the K.ii. Here, there is a dissatisfied landowner who stands to retain the defective structure built on his land, there arises the problem of unjust enrichment iii. The party who claims substantial performance has still breached the K and is liable for damages but in a lesser amount than a willful breachiv. Whether it was willful or not, willful forfeiture will preclude substantial performance

d. Plante v. Jacobsi. Substantial performance as applied to construction of a house does not mean that every

detail must be in strict compliance w/ the specifications and the plans…less than perfect is the test unless all details are made the essence of the K.ii. Diminished – Value Rule: difference between the value of the house as it stands w/ faulty and incomplete construction and the value of the house if it had been constructed in strict accordance w/ the plans and specificationsiii. Cost of Repair Rule: the cost to the non-breaching party to place himself in the same position he would have been in had the contract not been breachediv. Doctrine of substantial performance is applied when the unperformed portion does not destroy the purpose or value of the contract

2. The Perfect Tender Rule (commercially reasonable standard?)a. There is no room in commercial K for the doctrine of substantial performance; buyer is entitled to the goods as he asked for them

i. Seller must specifications perfectly or buyer can accept them, reject them all, or reject partb. UCC § 2-601: buyer should not be required to guess at his peril whether a breach is material; proof of materiality would sometimes require disclosure of the buyer’s private affairs such as secret formulas or processesc. UCC § 2-508: gives a seller the power to cure a defective tender – decision to attempt to cure is within the seller’s discretiond. UCC § 2-608: allows a buyer who has already accepted goods to revoke that acceptance (and return the goods to the seller) only if the “non-conformity substantially impairs value to him”e. UCC § 2-612: allows a buyer under a K for delivery of goods in installments to reject an installment only if a non-conformity as to the goods “substantially impairs the value of that installments” and to claim a breach of the whole K only for a breach that “substantially

impairs the value of the whole K”3. Divisibility

a. Gill v. Johnston Lumber Co.i. “If the part to be performed by one party consists of several and distinct items, and the price to be paid by the other is:

(a) apportioned to each item to be performed, or(b) is left to be implied by law, such a K will generally be held to be severable

ii. But if the consideration to be paid is single and entire the K must be held to be entire, although the subject of the K may consist of several distinct and wholly independent items.”

Page 22: Contracts II – Outline

b. IF a K can be equally apportioned and clear how much a party owes (even though still not fully performed) party can recover on work done

4. Restitutiona. Breaching party, even though at fault, should get restitution…a benefit had been conferred

i. The value of work done cannot exceed the K priceii. Bad Faith? Cts will look @ whether breach was willful or not – gen. can get restitution even though breach was willful

b. Non—Breaching party can choose damages or restitution (not capped)c. Britton v. Turner

i. A defaulting party, although unable to recover on his K, may recover under a quasi- contractual theory the reasonable value of his services less any damages to the other party arising out of default

d. Kirkland v. Archboldi. Based on a theory of unjust enrichment and the Ct’s abhorrence of forfeiture, a defaulting party may receive on a quasi-contractual theory for partial performance. ii. A defaulting party may recover for partial performance, on a quasi-contractual theory, the benefit received by the other party diminished by the damages to that other party arising from the default. iii. Can there be substantial performance on a divisible K? Yes. Substantial performance on that one part

D. Suspending Performance and Terminating the Contract1. Generally

a. Breach – i. Material

(a) Treat as a partial breach – right to damages; not a right to suspend or terminate the K(b) Treat as a total breach – suspend payment on K; give breaching party a chance to cure

ii. Non-material(a) Treat as a partial breach – continue with K, sue for damages

iii. Whether a breach is material or non-material is not up to the aggrieved partyiv. Whether to treat a material breach as partial or total is decided by the aggrieved party

2. After Acquired Evidencea. Walker & Co. v. Harrison

i. A part attempting to repudiate the K must convince the Ct the other party has materially breached the Kii. What is a material breach? (R1K § 275; R2K § 241)

(a) In determining the materiality of a failure fully to perform a promise the following circumstances are influential:

(1) the extent to which the injured party will be deprived of the benefit which he reasonably anticipated;

(2) the extent to which the injured party may be adequately compensated in damages for lack of complete performance

(3) the extent to which the party failing to perform…will suffer forfeiture;(4) the likelihood that the party failing to perform…will cure;(5) the extent to which the behavior of the party failing to perform…comports w/ standards of good faith and fair dealing

(b) Question of Factb. K&G Construction Co. v. Harris

i. Whenever possible, according to the intentions of the parties and the good sense of the case, mutual promises in a K will be regarded as dependent covenantsii. P’s duty to pay was dependent on D’s performance according to the K. P was entitled to withhold payment following D’s breach of his covenant to perform in a “work-manlike manner.”

Page 23: Contracts II – Outline

iii. D’s refusal to continue work was an additional breach for which P may recover its cost to completeiv. A party who is to perform work must perform according to the K before he is entitled to payment performance of work (according to K) is a constructive condition precedent to paymentv. If the refusal to pay an installment is justified on the owner’s part, the contactor is not justified in abandoning work by reason of that refusal…abandoning the work will be wrongful repudiation

3. Hindrance and Preventiona. Iron Trade Products Co. v. Wilkoff Co.

i. Mere difficulty of performance will not excuse a breach of K even though that difficulty was created by the other contracting party.

(a) P did not know it was interfering with D’s only source of supply(b) Had they known, D’s duty might have been discharged(c) D assumed the risk of the inflated market price and a deflated supply. However, they did not assume the risk that P would knowingly interfere with their part of the K

ii. “In the case of every K there is an implied undertaking on the part of each party that he will not intentionally and purposely do anything to prevent the other party from carrying out the agreement on his part.

b. New England Structures, Inc. v. Lorangeri. Where a party gives a reason for his conduct and decision touching…a controversy, he cannot, after litigation has begun, change his ground, and put his conduct upon …a different consideration. He is not permitted thus to mend his hold. He is estopped from doing it by a settled principle of law.ii. The grounds for breach would not be limited unless D showed it has detrimentally relied on the justification giveniii When a period of notice is required, the K remains in force and must continue to be performed during the established period set after notice of terminationiv. There is no waiver unless there is reliance; as long as no reliance, one can change his

defenseE. Prospective Non-Performance

1. Anticipatory Repudiationa. If, before the time of performance, a party to the K makes it clear by words or actions that she will breach when performance is due, anticipatorily repudiates the K

i. A clear, unequivocal, and voluntary repudiation by one of the parties is recognized as the equivalent of a material and total breach, provided that the threatened action or failure to act would be a material and total breach if it happened at the time due of performance

b. Purpose: i. Ability to mitigate damages; unfair to the other party for them to have to continue with their end of the bargain if she knows the first party will not perform; inefficientii. When one of the parties clearly repudiates a material promise in advance, the other may treat this as a breach immediately and may seek relief for breach w/o delay

c. Elementsi. The prospective action or inaction indicated by the Promisor must be serious enough to qualify as a material and total breach of the contractii. The Promisor’s statement or conduct must clearly indicate to the reasonable Promisee that the Promisor intends to breach materially when the time for performance arrives

(a) A Ct is unlikely to find a repudiation by conduct unless it is unquestionably so inconsistent w/ an intent to perform as promised, that the Promisor’s purpose of abandoning the K is beyond doubt.

iii. The Promisor’s statement or conduct in repudiating must be voluntary, that is, must have been deliberate and purposeful rather than inadvertent or beyond the Promisor’s control

Page 24: Contracts II – Outline

(a) Is the promisor saying she will not perform, or is she merely expressing a lack of confidence in her ability to perform, or complaining about the terms of the K?

d. Hochster v. De La Touri. A party to a K who renounces his intention to perform may not complain if the other party, instead of waiting until performance is due, elects to sue immediately for breach of K; the renunciation may be treated as a breach of K

(a) able to mitigate damages by not waitinge. Kanavos v. Hancock Bank & Trust Co.

i. Gen rule: when performance under a K is concurrent, one party cannot put the other in default unless he is ready, able, and willing to perform and has manifested this by some offer of performance…although tender is not necessary if the other party has shown that he cannot or will not performii. A party’s ability to recover for breach of a K giving a right of first refusal depends on his having had the financial resources to exercise the option

(a) P has the burden to prove that he can payf. McCloskey & Co. v. Minweld Steel Co.

i. In order to give rise to a renunciation amounting to a breach of K there must be an absolute and unequivocal refusal to perform or a distinct and positive statement of an inability to do so.ii. Though, where affirmative action is promised, mere failure to act, at the time when action has been promised, is a breach, failure to take preparatory action before the time when any performance is promised is not an anticipatory breach

2. Permissible Responses to Repudiationa. treat as an immediate breach; refuse to render own performance; and terminate the K, sue for reliefb. delay responding to the repudiation to see if the repudiating party repents

i. encourage the other party to repent by allowing an allotted time to recantii. party who allows the repudiating party to recant may change her mind at any time

before the retraction and accept the repudiationc. Retraction:

i. Promisor’s ability to retract is lost as soon as the promisee notifies the promisor that the repudiation has been acceptedii. Promisor cannot take back the repudiation if the promisee has treated it as final and has taken action in reliance on it

d. Cosden Oil & Chemical Co. v. Karl O. Helm Aktiengesellschaft i. Damages for a seller’s anticipatory repudiation are measured at a commercially reasonable time after the repudiation

(a) allows buyer time to chose his remedies and to consider mitigatione. United States v. Seacoast Gas Co.

i. A repudiation may be retracted up until the other party has commenced an action thereon or otherwise changed his positionii. UCC § 2-611(1) – until the repudiating party’s next performance is due he can retract his repudiation unless the aggrieved party has, since the repudiation, canceled or materially changed his position or otherwise indicated that he considers the repudiation final

3. Assurance of Due Performancea. UCC § 2-609 & R2K § 251 allows the promisee to demand assurance of due performance

i. § 2-609: if a party has reasonable grounds for insecurity regarding the other’s performnce, she may make a written demand for an adequate assurance of due performance; until that assurance is received, the party requesting it may, if commercially reasonable, suspend any of her own performance for which she has not already received the agreed return. The party receiving a justified demand for assurance must provide an adequate assurance within a reasonable time, not exceeding 30 days

Page 25: Contracts II – Outline

b. Pittsburgh—Des Moines Steel Co. v. Brookhaven Manor Water Co.i. UCC 2-609 allows a party to cease performance due to the repudiation of the other only where the repudiating party fails, upon demand, to give adequate assurance of payment

(a) The assurances to which PDM demanded were more than what they were entitled and the demand was not founded upon what in the Ct’s opinion was an actuating basis for the applicability of 2-609(b) Unreasonable for PDM to demand money in escrow as an assurance if they were willing to get paid within 30 beyond completion of the project according to the K(c) Because they didn’t have the money now, didn’t mean they weren’t to have it in the months to come(d) Asking for too much…Cts will treat as repudiation; they wanted more that what their K entitled them to (e) Need to be pretty insecure to ask for assurances

c. Narcon Power Partners v. Niagra Mohawk Power Corp.i. [In NY] A party to a K not involving the sale of goods does not have a right to demand assurance of future performance

V. Basic Assumptions: Mistake, Impracticability and FrustrationA. Mutual Mistake

1. Basicsa. Restatement § 152 – Mutual Mistake

i. Mistake goes to a basic assumption on which the K was madeii. Mistake has material effect on agreed exchange of performancesiii. Mistake is not one of which that party bears the risk

b. Unilateral Mistakei. Mistake of one party is on a basic assumption of the Kii. Mistake has a material effect on the agreed exchange of performancesiii. The other party knew or should have known about the mistake and took advantage of it; if so, the mistaken party may have the K rescinded

c. Steets v. Leonardi. If a man bind himself, by a positive, express K, to do an act in itself possible, he must

perform his engagement, unless prevented by the act of God, the law, or the other party to the K. No hardship, no unforeseen hindrance, no difficulty short of absolute impossibility, will excuse him from doing what he has expressly agreed to doii. No doctrine of mistake at the time, even though there was a mutual mistake here as to the quality of the soil the house what being built on.

d. Renner v. Kehli. Mutual mistake of fact is an accepted basis for rescission…mistake of material fact, which constitutes “an essential part and condition of the K.” (mistake on availability of water in land)ii. Absent proof of breach for fraud or misrepresentation, a party who rescinds a K may not recover consequential damages

(a) Restitution is available for both parties for any benefit conferred through either part performance or reliance – intended to avoid unjust enrichment not intended to be compensatory (return of any down payment and any improvements on the land; LO entitled to FMV of rent of land while P occupied the land)(b) Rescinding party must return any value received

2. Sale of Goodsa. Wood v. Boynton

i. Mistake was not mutual; both parties were unsure, so there was no mistake…they didn’t think it was something and it turned out to be anotherii. Seller took the risk of buying a worthless piece of rock; buyer was in a better position to find out what the rock was before she actually sold it

Page 26: Contracts II – Outline

iii. Ct said of only one possible ground for recovery… “that there was a mistake made by the vendor in delivering an article which was not the article sole, -- a mistake in fact as to the identity of the thing sold w/ the thing delivered.”

b. Sherwood v. Walkeri. A mutual mistake between the two parties; both seller and buyer thought the cow was barren. However, it turned out to be fertile and w/ calf.ii. Rose “was not in fact the animal, or the kind of animal, the defendants had intended to sell or the plaintiff to buy.”

3. The Risks of Limited Knowledgea. Rest. § 154(b) – a party bears the risk of mistake when “he is aware, at the time the K is made, that he has only limited knowledge w/ respect to the facts to which the mistake relates but treats his limited knowledge as sufficientb. “Conscious Ignorance” – one knows they ought to look into something but chooses not to they bear the risk, cannot rescind the K later

4. Mistake and Restitutiona. Where there is a mistake, the K may be avoided. Both parties are then entitled to restitution.

i. Modern view is that K may also be avoided where there is a mistake of lawB. Impracticability of Performance

1. Intro.a. Restatement § 261/265

i. Where, after a K is made, a party’s performance is made impracticable (is substantially frustrated)

ii. Without his fault by the occurrence of an event the non-occurrence of which was a basic assumption on which the K was made, iii. His duty to render that performance is discharged, unless the language or the circumstances indicate the contrary.

b. Impracticability change in the K after it was made / Mistake before the K was madei. PP involved in determining whether to excuse performance:

(a) K should be enforced as written(b) What would the parties had done had they thought about it?

c. Mineral Park Land Co. v. Howardi. A party is excused from performing under a K where such performance is so much more expensive than contemplated that it would be impracticable to complete.

(a) the difference in cost is so great…has the effect of making performance impracticable…the situation is not different from that of a total absence of earth and gravel

ii. A thing is impossible in legal contemplation when it is not practicable; and a thing is impracticable when it can only be done at an excessive and unreasonable cost.

2. Supervening Eventsa. Taylor v. Caldwell

i. In Ks in which the performance depends on the continued existence of a given person or thing, a condition is implied that the impossibility of performance arising from the perishing

of the person or thing shall excuse the performanceii. Subject to the implied condition that the parties shall be excused in case, before breach, performance becomes impossible from the perishing of the thing w/o fault of the contractoriii. The excuse from the K’s performance is implied in law because from the nature of the K is it apparent it was made on the basis of the continued existence of the particular, specified thing

3. Under UCC Art. 2a. UCC § 2-613 – a K is “avoided” if goods that are required for performance of the K suffer a “casualty” w/o the fault of either party before the risk of loss passes to the buyer

Page 27: Contracts II – Outline

i. Limited to situations in which the goods that suffer the casualty are the only goods that would satisfy the K. If does not apply when the seller could satisfy its obligation by tendering goods other than those destroyed ii. Analogous w/ c/l doctrine of impossibility

b. UCC § 2-615 provides relief when the seller’s performance “has been made impracticable by the occurrence of a contingency the non-occurrence of which was a basic assumption on which the K was made

i. (a) delay is not a breach if impracticable (contingent on basic assumption) … (c) must notify the buyerii. Standard for impracticability is high – a mere increase in cost is not sufficient

c. UCC § 2-614 provides relief when the agreed method of delivery becomes commercially impracticable

i. (1) commercially reasonable substitute must be tendered and acceptedd. Transatlantic Financing Corp. v. United States

i. When the issue of impossibility is raised, the Ct must construct a condition of performance based on changed circumstances involving the following:

(a) A contingency, something unexpected, must have occurred,(1) The usual rout was impossible, however, the alternate rout was not impracticable (legally impossible)(2) To justify relief there must be more of a variation between expected cost and the cost of performing by alternative means than was present here

(b) The risk of unexpected occurrence must not have been allocated either by agreement or custom, and

(1) Proof that the risk of a contingency’s occurrence has not been allocated may be expressed in or implied from the agreement(2) Such proof may also be found in the surrounding circumstances, including custom and usages of the trade

(c) Occurrence of the contingency must have rendered performance commercially impracticableii. A Ct must find these three requirements satisfied, or the plea of impossibility must failiii. In determining impossibility, the Ct will look first to see which party assumed the risk of unforeseen circumstances.

(a) Knowledge of the [crisis] will tend to show assumption of the risk (b) The Ct would not place a burden on one party to preserve the other’s profit(c) Foreseeability is used as a tool in considering where the risk was to be allocated

4. Foreseeabilitya. If the risk is foreseeable, party assumes an obligation to performb. Usually the Ct will assign the risk to the superior risk bearer ---the one who is in a better position to assess the risk

5. Risk—Bearing Analysisa. Selland Pontiac—GMC, Inc. v. King

i. A partial failure of a seller’s source of supply generally ahs been treated as a foreseeable contingency, the risk of which is allocated to the seller absent a specific provision to the

contrary in the Kii. Supply of the product was a basic assumption on which the K was made. These became impracticable to supply when S ceased manufacturing

(a) Seller’s supplier was specified in the K(b) King did not expressly assume the risk of Superior’s ceasing production

b. Restatement § 261 – Even if a party contracts to render a performance that depends on some act by a third party, he is not ordinarily discharged because of a failure by that party because this is also a risk that is commonly understood to be on the obligor

Page 28: Contracts II – Outline

6. Force Majeure Clausesa. A party anticipates one or more events that it cannot readily prevent and that might impede its performance; b. a FM clause excuses a party from performance if that performance is rendered impracticable as

a result of such an uncontrollable event7. Casualty to Goods

a. UCC § 2-613 – avoidance of K may occur by reason of a casualty before the risk of loss passes “in a proper case under a ‘no arrival, no sale’ termb. If the goods are “indentified” risk passes to the buyerc. Eastern Air Lines, Inc. v. Gulf Oil Corporation

i. The party undertaking the burden of establishing “commercial impracticability” by reason of allegedly increased raw material costs undertakes the obligation of showing the extent to which he suffered, or will suffer, losses in performing his Kii. If a contingency is foreseeable, it and its consequences are taken outside the scope of UCC 2-615, because the party disadvantaged by fruition of the contingency might have protected

himself in his KC. Frustration of Purpose

1. In Generala. Requirements:

i. event substantially frustratedii. a basic assumption of the Kiii. no fault of party seeking to be excusediv. no assumption of greater obligation

b. Krell v. Henryi. Where the object of one of the parties (coronation of the king) is the basis upon which both parties K, the duties of performance are constructively conditioned upon the attainment of

that object(a) Inferred from the surrounding circumstances the purpose of the K, and that was the

foundation of the K – with the defeat of the purpose of the K, the performance is excused

ii. Performance is not impossible; a constructive condition based on the attainment of the purpose or object has arisen; the value of performance to the party receiving it turns out to be significantly different than expected

c. Swift Canadian Co. v. Baneti. Assuming a party is ready to perform, he is not obligated to perform in vain where the other party has given notice of refusal to accept performanceii. F.O.B. means that seller’s responsibility terminates at the location specified; goods were “identified” and risk of loss had passed to buyeriii. Wasn’t frustration of purpose because buyer had the obligation once the pelts were on board (Gov’t regulation frustrated purpose of having pelts in Philly)iv. UCC 2-615(a) expressly allows impracticability due to gov’t intervention (case was before UCC)

d. Chase Precast Corp. v. John J. Paonesssa Co.i. Under frustration, performance remains possible but the expected value of the performance to the party seeking to be excused has been destroyed by the fortuitous eventii. A defendant may rely on frustration of purpose as a defense to a breach of contract claim if the risk of the occurrence of the frustrating event is not allocated by the K to the defendant.

(a) Chase was held to a higher standard due to knowledge they supposedly hade. Northern Indiana Public Service Co. v. Carbon County Coal Co.

i. A party cannot avoid performance of a contract on the basis of impracticability or impossibility where the K specifically shifts the risk of such to that party

(a) The risk was allocated to NIPSCO from the K

Page 29: Contracts II – Outline

(b) K being a fixed price K is an explicit assignment of the risk of market price decreases to the buyer

ii. Force Majeure clause excuses performance where gov’t acts prohibit the utilization of coal(a) Use was uneconomical, not illegal

D. Half Measures1. Impracticability and Reliance

a. What happens when a K is avoided? Damages?i. Divisibility is available;ii. Restitution is available (for work done on performance of K)iii. Reliance damages are not available (available if no other damages to award and to avoid an injustice)iv. Deposits are returned; unjust enrichment if not (minus restitution for the other party)

b. Young v. City of Chicopeei. There is an implied condition that a thing upon which work is to be done shall continue to exist, and if it is destroyed w/o fault, the owner is liable for the value of the work done.ii. “Wrought-in” rule: not liable for the lumber not incorporated into the bridge; lumber was still in Young’s control, just there to ensure rapid constructioniii. Repair K are different than construction K

(a) under construction K, contractor would not be compensated upon loss for destruction(b) under repair K, contractor is excused from further performance and is compensated for that which he has done

(1) The continued existence of the structure being repaired is a basic assumption of the parties

VI. Third Parties: Rights and ResponsibilitiesA. Third—Party—Beneficiary Contracts

1. Restatement Formulasa. Restatement 302—Intended Beneficiaries

i. Claimant is intended beneficiary if recognition is appropriate to effectuate parties intentions and if claimant meets one of two qualifications:

(a) Creditor of promisee, or (to pay money)(b) Intent of promisee to give benefit to claimant

b. Incidental Beneficiariesi. A third party who may enjoy an advantage through the performance of a K, but has no

enforceable interest in its performanceii. Ineligible to enforce a K between other parties, even though they would benefit from its performanceiii. Recognition of a right to performance in the claimant is appropriate to effectuate the intention of the contracting parties

b. Lawrence v. Foxi. A third party for whose benefit a contract is made may bring an action for its breach

(a) Lawrence was a “creditor” beneficiary(b) R2K § 133 has eliminated the creditor/donee distinction

c. Seaver v. Ransoni. A niece for whose benefit a promise was made to her aunt may successfully bring an action for breach of that promiseii. Four situations of third—party beneficiaries: Creditors; Close Family; Public K; Promise to

beneficiaryd. Detroit Institute of Arts Founders Society v. Rose

i. Two parties may enter into a contract to benefit a third party beneficiary who is then entitled to enforce contractual obligations without being a party to the K and thus may sue the obligor for breach.

Page 30: Contracts II – Outline

ii. To be valid, there need not be express language in the K creating a direct obligation to the third party beneficiary.iii. However, a K can only result in an obligation to a third party if both parties to the K intended to create a direct obligation form the promisor to the third party

(a) Intent is to be determined form the terms of the K read in light of the circumstances attending the making of the K, including the motives and the purposes of the parties

2. Distribution Networksa. A distributor of goods holding an exclusive marketing license has portrayed itself as a third— party beneficiary of an agreement between its supplier and another licensee

3. Contracts for Public Servicesa. Rathke v. Corrections Corporation of America, Inc.

i. A third party will be deemed to be an intended beneficiary of a K where the K references and incorporates verbatim language from a different K between the promisor and the third party that benefits the third party.ii. Maj. rule: there is no requirement of mutual intent, as to the right of enforcement, on the part of the contracting parties; iii. Instead, it is the intent or purpose of the promisee, which pays for the promise that has been generally considered as governingiv. Restatement view: a party may be deemed an intended beneficiary regardless of the actual intentions of the parties—based on the objective theory of K; especially promisee’s motive

4. Third—Party Claims for Bad—Faith Breacha. Verni v. Cleveland Chiropractic College

i. A third party is not an intended beneficiary of a K where the terms of the K do not directly and clearly express the intent to benefit the third party or any class to which the party belongs, even if the third party is incidentally benefitted by the K.

5. Attorney—Client Contracts and Third Partiesa. Grigerik v. Sharpe

i. The intent of both parties to a contract determines whether a third party has contract rights as a third-party beneficiary

6. Vesting of Third-Party Rights – Restatement 311(3)a. Contracting parties can no longer discharge or modify a duty owned to an intended beneficiary once he has manifested assent to the promise at the request of one of the parties, or has brought suit on the promise, or has changed his position materially in justifiable reliance on it, and has so acted before receiving notification of the discharge or modification.b. You can change your mind about making someone a third-party beneficiary—must do so before the rights vest

B. Delegation of Duties1. What is the difference between an assignment of rights and a delegation of duties?

a. A right is the flip side of a dutyb. Assign a right (a right for the other party to perform it on your behalf)c. Delegate a duty (to do something)

i. you are not off the hook once you delegate a duty; need the other party’s written consent (novation—other party’s written assent)ii. Delegor remains liable

2. When is a duty non-delegable?a. Personal services; b. UCC 2-210(1); Rest. 318c. Whether the obligee has a substantial interest in having the promisor perform.

(a) Customer more likely to object to an assignment (delegation of a duty)(1) Manufacturer gets money—interchangeable

(b) Object to a delegation to pay money—not as financially in as good a position as the delegor was

Page 31: Contracts II – Outline

d. Sally Beauty Co. v. Nexxus Products Co.i. The duty of performance under an exclusive distributorship may not be delegated to a competitor in the marketplace or to the wholly owned subsidiary of a competitor w/o the obligee’s consent.ii. Straight stock purchase – no assignment; change of control—get around a “no assignment” provision by selling the co.; Merging co. –creating a new entity, changing the actual party to the K, a real assignment

C. Assignment of Rights1. Assignability of Rights: Means of Assignment

a. Rights may be assigned, unless assignment will materially change the dutyb. Entitles demands for assurances from assigneec. Herzog v. Irace

i. An assignment is binding upon the obligor where the assignor has intended to relinquish his rights and the obligor is notified of the intent to relinquish his rights to the assignee.ii. Assignment must not contain any control over the right assigned or any power of revocation; once obligor has notice of the assignment, the fund becomes a trust

d. Language Required to make an assignment effectivei. Has to be a present transfer; can’t retain power of interest

(a) A future interest? Yesii. Based on all circumstances: words, and conductiii. Can have a legal effect w/o an assignment

e. Usually an agreement to pay out of a particular fund is not acceptable2. Gifts

a. Assignment of a gift w/o a writing (not the subj. of ST of F)b. Can revoke whenever you want, there’s no consideration